27

If $a,b,c$ are the sides of a triangle inscribed in a circle of radius $R$. Is it true that

$$ \frac{abc}{R^3} \ln \left(\frac{a}{R}\right)\ln\left(\frac{b}{R}\right)\ln \left(\frac{c}{R}\right) > - \ln 2 \tag 1 $$

I ran a Monte Carlo simulation for the expression on the LHS and obtained a minima of $- 0.6923259$ after a billion samples. This value is slightly greater than $\ln2 \approx 0.6931$. Can this proved?

Update 1: Using the insight from the comment made by @Anon we can show the slightly weaker result that LHS $> - \frac{4\ln^2 2}{e} \approx -0.7069951$. If one side of the triangle is less than $R$ and the other two sides are greater than $R$ then the LHS will be negative. Hence we need to minimize $x \ln x$ for one side and maximize it for the other two sides. The minimum value of $x\ln x$ occurs at $x = \frac{R}{e}$ and for the triangle of circum radius $R$, since no side can exceed $2R$ hence $x\ln x < \frac{2R}{R} \ln \frac{2R}{R} = 2\ln 2$. Hence the LHS of $(1)$ must be at least $> - \frac{4\ln^2 2}{e}$.

Update 2: An improvement of the estimate in update 1 is as follows. If one side of a triangle is $a = \frac{1}{e}$, what is the maximum possible length of the other sides. Simple algebraic manipulations give $b = c = \sqrt{2 + \sqrt{4 - \frac{1}{e^2}}}$ and substituting these in $(1)$ we get

$$ LHS \ge -\frac{1}{e}\left(\sqrt{1 - \frac{1}{2e}} + \sqrt{1 + \frac{1}{2e}}\right)^2 \ln^2 \left(\sqrt{1 - \frac{1}{2e}} + \sqrt{1 + \frac{1}{2e}}\right) \approx -0.692325 $$

But must be noted that this may not be the minima since the true minima may occur at $a \ne \frac{1}{e}$.

Update 3: Under the assumption that the minima will occur when the triangle is isosceles, we can prove that the minima is $-0.62918$ as shown by K.defaoite. I have separate and proof which I have posted below.

TShiong
  • 1,270
  • Isn't this trivially true? Let a/R = x, b/r = y, c/R = z. Then you want to show that $f(x) f(y) f(z) > - \ln 2$ where f(t) = t ln t. But the minimum of f is -1/e, and $(-1/e)^3 > - \ln 2$ – Anon Apr 19 '23 at 14:23
  • Yikes, I forgot one of the three values could be positive and greater than $\frac{1}{e}$, thereby lowering the minimum. Ignore the above comment. – Anon Apr 19 '23 at 14:25
  • @Anon Moreover, we have the additional restriction of the circumradius. With your transformation, the circumradius is equal to $1/e$ but it must be $1$ as per the question. – Nilotpal Sinha Apr 19 '23 at 14:26
  • 2
    Could you say which kind of triangle (in your simulation) gives the extremal value ? It could give a previous indication... – Jean Marie Apr 19 '23 at 15:20
  • @JeanMarie The minima always occurs when the triangle is nearly isosceles with two sides approaching $2R$ and the third side approaching $R/e$. I have added the explanation for this on the post. – Nilotpal Sinha Apr 20 '23 at 02:13
  • If we assume $R=1$ and the triangle is isosceles, then the minimum value of $\frac{abc}{R^3} \ln \frac{a}{R}\ln \frac{b}{R}\ln \frac{c}{R}$ is the minimum value of $(2\cos{x})^2 (2\sin{2x})(\ln{(2\cos{x})})^2(\ln{(2\sin{2x})})$ which is approximately $-0.69291818$. – Dan Apr 20 '23 at 05:32
  • @Dan You are right, the closed form was very close to the minima but not quite. – Nilotpal Sinha Apr 20 '23 at 05:51

4 Answers4

6

Proof.

We need to prove that, for all $x, y, z > 0$ with $x + y + z = \pi$, $$8\sin x \sin y \sin z \cdot \ln (2\sin x) \cdot \ln(2 \sin y) \cdot \ln(2\sin z) > -\ln 2. \tag{1}$$

We only need to prove the case that $\ln (2\sin x) \cdot \ln(2 \sin y) \cdot \ln(2\sin z) < 0$.

We split into two cases.

Case 1: $\ln(2\sin x) < 0, \ln(2\sin y) < 0, \ln(2\sin z) < 0$

It is easy to prove that $-\mathrm{e}^{-1} \le u\ln u < 0$ for all $u\in (0, 1)$. Thus, we have $-\mathrm{e}^{-1} \le 2\sin x \ln(2\sin x) < 0$ etc. and $$\mathrm{LHS} \ge (-\mathrm{e}^{-1})^3 > -\ln 2.$$

$\phantom{2}$

Case 2: $\ln(2\sin x) > 0, \ln(2\sin y) > 0, \ln(2 \sin z) < 0$

We have $x, y \in (\pi/6, 5\pi/6)$ and $z \in (0, \pi/6)$.

Note that $x\mapsto \ln (2\sin x)$ is concave on $(\pi/6, 5\pi/6)$. We have $$\ln(2\sin x) + \ln(2\sin y) \le 2\ln\left(2\sin \frac{x + y}{2}\right). \tag{2} $$ Using (2), we have $$2\sin x \cdot 2\sin y \le \left(2\sin \frac{x + y}{2}\right)^2. \tag{3}$$ By AM-GM, using (2), we have $$ \ln(2\sin x) \ln (2\sin y) \le \frac14[\ln(2\sin x) + \ln(2\sin y)]^2\le \ln^2\left(2\sin \frac{x + y}{2}\right). \tag{4}$$

From (1), (3) and (4), it suffices to prove that $$ \left(2\sin \frac{x + y}{2}\right)^2\cdot \ln^2\left(2\sin \frac{x + y}{2}\right) \cdot 2\sin z \cdot \ln(2\sin z) > -\ln 2$$ or $$ \left(2\cos\frac{z}{2}\right)^2\cdot \ln^2\left(2\cos \frac{z}{2}\right) \cdot 2\sin z \cdot \ln(2\sin z) > -\ln 2$$ or $$\ln^2 2 > \left(2\cos\frac{z}{2}\right)^4\cdot \ln^4\left(2\cos \frac{z}{2}\right) \cdot 4\sin^2 z \cdot \ln^2(2\sin z)$$ or $$\ln^2 2 > (2 + 2\cos z)^2 \cdot \frac{1}{2^4}\ln^4 (2 + 2\cos z)\cdot 4(1 - \cos^2 z)\cdot \frac{1}{2^2}\ln^2(4 - 4\cos^2 z)$$ where we use $4\cos^2 \frac{z}{2} = 2 + 2\cos z$.

Letting $u = \cos z$, it suffices to prove that, for all $\frac{\sqrt 3}{2} < u < 1$, $$\ln^2 2 > (2 + 2u)^2 \cdot \frac{1}{2^4}\ln^4 (2 + 2u)\cdot 4(1 - u^2)\cdot \frac{1}{2^2}\ln^2(4 - 4u^2) \tag{5}$$ which is true. The proof is given at the end.

We are done.

$\phantom{2}$


Proof of (5):

Using $\frac{64}{63}u^2 + \frac{63}{64} - 2u = \frac{1}{4032}(64u-63)^2 \ge 0$, we have $$(2 + 2u)^2 = 4 + 4u^2 + 8u \le 4 + 4u^2 + 4\left(\frac{64}{63}u^2 + \frac{63}{64}\right) .$$

It suffices to prove that $$\ln^2 2 > \frac{1}{256}\left(\frac{508}{63}u^2 + \frac{127}{16}\right)\ln^4 \left(\frac{508}{63}u^2 + \frac{127}{16}\right) \cdot (1 - u^2)\ln^2(4 - 4u^2). \tag{6}$$

Letting $4 - 4u^2 = v$, it suffices to prove that, for all $v \in (0, 1)$, $$\ln^2 2 > \frac{1}{1024}\left(\frac{16129}{1008} - \frac{127}{63}v\right) \ln^4 \left(\frac{16129}{1008} - \frac{127}{63}v\right) \cdot v\ln^2 v. \tag{7}$$

We can prove that, for all $v\in (0, 1)$, $$v\ln^2 v \le \frac98\ln^2 2 + (9\ln^2 2 - 6\ln 2)(v - 1/8). \tag{8}$$ (Note: The RHS is the first order Taylor approximation of LHS around $v = 1/8$. Proof: Take derivative.)

We can also prove that, for all $v\in (0, 1)$, $$\ln \left(\frac{16129}{1008} - \frac{127}{63}v\right) \le \ln\frac{15875}{1008} - \frac{16}{125}(v - 1/8). \tag{9}$$ (Note: The RHS is the first order Taylor approximation of LHS around $v = 1/8$. Proof: Take derivative.)

It suffices to prove that, for all $v\in (0, 1)$, \begin{align*} \ln^2 2 &> \frac{1}{1024}\left(\frac{16129}{1008} - \frac{127}{63}v\right) \left[\ln\frac{15875}{1008} - \frac{16}{125}(v - 1/8)\right]^4\\[10pt] &\qquad \times \left[\frac98\ln^2 2 + (9\ln^2 2 - 6\ln 2)(v - 1/8)\right]. \tag{10} \end{align*}

Using $\ln\frac{15875}{1008} < \frac{397}{144}$, it suffices to prove that for all $v\in (0, 1)$, \begin{align*} \ln^2 2 &> \frac{1}{1024}\left(\frac{16129}{1008} - \frac{127}{63}v\right) \left[\frac{397}{144} - \frac{16}{125}(v - 1/8)\right]^4\\[10pt] &\qquad \times \left[\frac98\ln^2 2 + (9\ln^2 2 - 6\ln 2)(v - 1/8)\right] \end{align*} or \begin{align*} 1 &> \frac{1}{1024}\left(\frac{16129}{1008} - \frac{127}{63}v\right) \left[\frac{397}{144} - \frac{16}{125}(v - 1/8)\right]^4\\[10pt] &\qquad \times \left[\frac98 + \left(9 - \frac{6}{\ln 2}\right)(v - 1/8)\right]. \tag{11} \end{align*}

We split into two cases.

Case 1: $0 < v \le 1/8$

Using $\ln 2 > \frac{70}{101}$, it suffices to prove that \begin{align*} 1 &> \frac{1}{1024}\left(\frac{16129}{1008} - \frac{127}{63}v\right) \left[\frac{397}{144} - \frac{16}{125}(v - 1/8)\right]^4\\[10pt] &\qquad \times \left[\frac98 + \left(9 - \frac{6}{70/101}\right)(v - 1/8)\right].\tag{12} \end{align*} Letting $v = \frac{1}{8 + w}$ for $w \ge 0$, (12) is written as $$\frac{F(w)}{10113169489920000000000000(8+w)^6} > 0$$ where \begin{align*} F(w) &= 2416264063255035327931w^6 + 53231769508888881467040w^5\\ &\qquad + 440259554245249137560000w^4 + 4667308406207884160000000w^3\\ &\qquad + 67053001632350760000000000w^2 + 463246417085920000000000000w\\ &\qquad + 1111827500508480000000000000. \end{align*} Thus, (12) is true.

Case 2: $1/8 < v < 1$

Using $\ln 2 < \frac{61}{88}$, it suffices to prove that \begin{align*} 1 &> \frac{1}{1024}\left(\frac{16129}{1008} - \frac{127}{63}v\right) \left[\frac{397}{144} - \frac{16}{125}(v - 1/8)\right]^4\\[10pt] &\qquad \times \left[\frac98 + \left(9 - \frac{6}{61/88}\right)(v - 1/8)\right]. \tag{13} \end{align*} Letting $v = \frac{1 + 8w}{8 + 8w}$ for $w > 0$, (13) is written as $$\frac{G(w)}{17625809682432000000000000(1+w)^6} > 0$$ where \begin{align*} G(w) &= 823425037465312092776856w^6 + 3524126527531263109253089w^5\\ &\qquad +5778219879301331456143375w^4+4426205206524727153031250w^3\\ &\qquad +1504381676597156324218750w^2+163085374636585205078125w\\ &\qquad +7391954425377685546875. \end{align*} Thus, (13) is true.

We are done.

River Li
  • 49,125
  • 1
    I think, we are "not" done. Numbers that you suddenly invent for no explanation and don't look pretty are not instructive at all. If there's an original solution to this problem, your answer probably isn't. That doesn't mean your answer isn't good. Because, we also don't know what the original solution was. (PS: I am not a downvoter ) – User Apr 21 '23 at 13:16
  • 4
    @User Happy to see your last sentence! Thanks for comment. First, this is not an solution by hand. So, it is easy to verify everything by using computer. Second, as a solution, I think we don't need to include what motivate the solution. For example, we use computer to find an SOS expression. But this SOS expression itself is easily verified by hand! So in this case, it is a solution even it is very hard to find it by hand. – River Li Apr 21 '23 at 13:22
  • 1
    @User Can you give a specific point which is not good. Let us discuss it. – River Li Apr 21 '23 at 13:24
  • 1
    I wasn't talking about mathematical invalidity. The points were different. – User Apr 21 '23 at 14:16
  • 1
    @User I don't understand. Do you mean e.g. how $v = 1/8$ comes from? I think there is no need to include how I found $1/8$. People simply verify its truth, e.g., $\ln \left(\frac{16129}{1008} - \frac{127}{63}v\right) \le \ln\frac{15875}{1008} - \frac{16}{125}(v - 1/8)$ is true. – River Li Apr 21 '23 at 14:34
  • 2
    @User Please see my answer for this question: https://math.stackexchange.com/questions/449755/prove-that-a-d2b-c2-geq-1-6-if-a24b2-4-and-cd-4/4539078#4539078. I gave the construction $(a - d)^2 + (b - c)^2 + \frac12(a^2 + 4b^2 - 4) - \frac{2\sqrt 2}{3}(cd - 4)$. There is no need to include how I found (explanations) the numbers $\frac12$ and $- \frac{2\sqrt 2}{3}$. People simply verify its true. It is indeed a proof. That's enough. – River Li Apr 21 '23 at 14:38
  • 2
    Wow that's pretty hardcore! (+1) Would you mind giving some general methodology for coming up with these bounds? Perhaps a link would suffice. – Trebor Apr 24 '23 at 10:36
  • 2
    @Trebor Thanks. I used computer to know that the maximum of RHS of (5) occurs near $\frac{63}{64}$. So in the proof of (5), I first convert $u$ to $u^2$ using $\frac{64}{63}u^2 + \frac{63}{64} - 2u = \frac{1}{4032}(64u-63)^2 \ge 0$, so RHS in (5) is a function of $u^2$. Second, with the substitution $4 - 4u^2 = v$, I found that the maximum of RHS of (7) occurs near $v = 1/8$. Then I used linear approximation of the functions in RHS of (7). – River Li Apr 24 '23 at 12:07
  • 5
    What an effort! – K.defaoite Apr 24 '23 at 18:10
5

Partial answer / simplification.

WLOG consider $R=1$. Other values of $R$ follow by simple scaling.

Consider a unit circle just touching the origin, i.e consider the set $\{(x,y)\in\Bbb R^2\mid \Vert(x-1,y)\Vert=1\}$. Again, WLOG, we can consider one of the points to be precisely at the origin $(0,0)$, as all the other cases follow by a simple rotation.

So, we have reduced our four-parameter problem (a,b,c,R) into a two-parameter problem. Next, any triangle in the unit circle can be described by three points. The points are $$P_0=(0,0) \\ P_1=(1+\cos\alpha,\sin\alpha) \\ P_2=(1+\cos\beta,\sin\beta) \\ \text{where}~\alpha,\beta\in(-\pi,\pi) ~\text{and}~\alpha\neq \beta$$ Getting the side lengths is easy. We call $$a=\Vert P_1-P_0\Vert=2\cos(\alpha/2) \\ b=\Vert P_2-P_0\Vert=2\cos(\beta/2) \\ c=\Vert P_2-P_1\Vert=\sqrt{2-2\cos(\alpha-\beta)}$$

Therefore, we can equivalently seek $$\inf_{|\alpha|<\pi \\ |\beta|<\pi}\left\{4\cos(\alpha/2)\cos(\beta/2)\sqrt{2-2\cos(\alpha-\beta)}~\log(2\cos(\alpha/2))\log(2\cos(\beta/2))\log\sqrt{2-2\cos(\alpha-\beta)}\right\} \\ =\inf_{|\alpha|<\pi \\ |\beta|<\pi}\left\{f\big(2\cos(\alpha/2)\big)f\big(2\cos(\beta/2)\big)f\left(\sqrt{2-2\cos(\alpha-\beta)}\right)\right\}$$

Putting this minimization problem into Mathematica using the command

f[z_] := z*Log[z]
NMinimize[{f[2*Cos[\[Alpha]/2]]*f[2*Cos[\[Beta]/2]]*
   f[Sqrt[2 - 2 Cos[\[Alpha] - \[Beta]]]], \[Alpha] \[Element] 
   Reals, \[Beta] \[Element] Reals, Abs[\[Alpha]] < \[Pi], 
  Abs[\[Beta]] < \[Pi]}, {\[Alpha], \[Beta]}]

It returns the value of $-0.692918$ at $\alpha=-\beta=0.177529$, which is just barely bigger than $-\log 2$.

I think if you can prove that the minimum has to occur on the line $\alpha=-\beta$, then you should be able to prove that the minimum is definitely larger than $-\log 2$. Maybe the best way to do that is to let $$F(x,y)=f(2\cos(x/2))f(2\cos(y/2))f(\sqrt{2-2\cos(x-y)})$$ Explicitly calculate $\nabla F$, and show that the only suitable solutions to $\nabla F(x,y)=0$ are on the line $\alpha=-\beta$.

K.defaoite
  • 13,890
  • This is good start. It would be interesting to see if $\ln 2$ is a numerical con-incidence or a consequence of some underlying constructions. – Nilotpal Sinha Apr 20 '23 at 02:54
3

Let $a,b,c$ be the sides of a triangle and $R$ be its circumradius. Let $f\left(\frac{a}{R}, \frac{b}{R}, \frac{c}{R}\right)$ whose maxima or minima we are interested. WLOG we can take $R = 1$ since other values of $R$ follow by scaling. Thus we are interested in the extreme values of $f(a,b,c)$.

Assumption: If $f(a,b,c)$ is symmetric then the extreme values will occur when the triangle is isosceles.

I do not have a proof of this but intuitively this seems plausible since for a symmetric function, interchanging any two sides does not alter the value of $f$. In many cases, the extreme values occur when all three sides are equal but equilateral triangles are a subset of isosceles triangles so it does not violate the above assumption.

Let $a$ be the base of the isosceles triangle which gives the extreme values and let the other two equal sides be of length $x$ each. Since the circumradius $R = 1$ using equation 1 in this link we have

$$ \frac{x^2}{\sqrt{(2x+a)(2x-a)}} = 1 \tag 1 $$

Solving for $x$ we get $x = \sqrt{2 \pm \sqrt{4-a^2}}$ as the feasible solutions. Hence the extreme values of $f(a,b,c)$ will occur at $f\left(a, \sqrt{2 \pm \sqrt{4-a^2}}, \sqrt{2 \pm \sqrt{4-a^2}}\right)$. Since $a$ is the side of a triangle inscribed in circle of radius $R=1$, the extreme values will be the local maxima and minima for $0 \le a \le 2$. Plugging these values of $a,b,c$ in $f(a,b,c) = abc \ln a \ln b \ln c$, the minimum value is the local minima of

$$ (a\ln a) \left(\sqrt{2 \pm \sqrt{4-a^2}}\right)^2 \ln \left(\sqrt{2 \pm \sqrt{4-a^2}}\right) \tag 2 $$

for $a \le 2$ which is $-0.692918$ at $a = 0.353196$ as calculated by Wolfram Alpha.

enter image description here

Additionally solving $(2)$ for maxima gives the maximum value of $0.861242$ at $a = b = c = \sqrt{3}$ i.e. equilateral triangle

  • 1
    Wolfram gives more decimals when taking the derivative first. https://www.wolframalpha.com/input?i=0%3D%28log%282+%2B+sqrt%284+-+a%5E2%29%29+%28%284+-+a%5E2+%2B+2+sqrt%284+-+a%5E2%29%29+log%282+%2B+sqrt%284+-+a%5E2%29%29+-+2+log%28a%29+%28a%5E2+%2B+%28-2+%2B+a%5E2+-+sqrt%284+-+a%5E2%29%29+log%282+%2B+sqrt%284+-+a%5E2%29%29%29%29%29%2F%284+sqrt%284+-+a%5E2%29%29 yields a ≈ 0.353196488039895365948597762399265372962425249950610743788561092920118961451905685559296104978385935... and min=--0.692918181515209899964063294317895428118788002581059785480776968491758297938218661727502579969330291033 – Wolfgang Apr 21 '23 at 10:16
3

Does every triangle satisfy

$$ \frac{abc}{R^3} \ln \frac{a}{R}\ln \frac{b}{R}\ln \frac{c}{R} > - \ln 2 $$

where $R$ is the circumradius?

Substituting $\frac{a}{R}=2\sin\alpha, \frac{b}{R}=2\sin\beta, \frac{c}{R}=2\sin\gamma$ this becomes

$$ 8\sin\alpha\sin\beta\sin\gamma\ln(2\sin\alpha)\ln(2\sin\beta)\ln(2\sin\gamma) > - \ln 2 $$

Replacing $\gamma=\pi-(\alpha+\beta)$ gives

$$ 8\sin\alpha\sin\beta\sin(\alpha+\beta)\ln(2\sin\alpha)\ln(2\sin\beta)\ln(2\sin(\alpha+\beta)) > - \ln 2 $$

So this reduces to finding the minimum value of a function

$$ f(x,y)=8\sin(x)\sin(y)\sin(x+y)\ln(2\sin(x))\ln(2\sin(y))\ln(2\sin(x+y)) $$ under the restraint $0<x+y<\pi$.

Wolfram Alpha computes a minimum value of \begin{eqnarray}f_\min&=&-.6929184\\ &=&\ln(0.5001144034)\\ &=&-\ln(2)+0.000285555 \end{eqnarray}at $(x,y)\approx(0.177529,1.48203)$

So it seems that the answer is yes.